Fix solution rounding in Serway and Jewett v8's problem 27.1.
authorW. Trevor King <wking@tremily.us>
Mon, 30 Apr 2012 19:33:30 +0000 (15:33 -0400)
committerW. Trevor King <wking@tremily.us>
Mon, 30 Apr 2012 19:33:30 +0000 (15:33 -0400)
latex/problems/Serway_and_Jewett_8/problem27.01.tex

index 27bbf432ddcc77b5c8a5aa90c91741f31a6636f2..2e738ba4e8d86d2b1c7f333345047de19b48aa10 100644 (file)
@@ -13,6 +13,6 @@ where $q=1.60\E{-19}\U{C}$ is the charge of a single proton.  The
 number of strikes in the allotted time is thus
 \begin{equation}
   \Delta N = \deriv{t}{N} \cdot \Delta t = \frac{I\Delta t}{q}
-    = \ans{17.9\E{15}\U{protons} = 17.9\U{petaprotons}} \;.
+    = \ans{18.0\E{15}\U{protons} = 18.0\U{petaprotons}} \;.
 \end{equation}
 \end{solution}